普通物理梁斌6^10答案

更新时间:2023-10-07 12:32:01 阅读量: 综合文库 文档下载

说明:文章内容仅供预览,部分内容可能不全。下载后的文档,内容与下面显示的完全一致。下载之前请确认下面内容是否您想要的,是否完整无缺。

习题6

6-1有一个弹簧振子,振幅为2?10?2m,周期为1s,初相为3?/4. 试写出它的振动方程,并画出x-t图、?-t图和a-t图.

解:??2??2?s?1; T3振动方程:x?2?10?2cos(2?t??)m;

43速度:v??4??10?2cos(2?t??)m?s?1;

43加速度:a??8?2cos(2?t??)m?s?2。图略

46-2谐振动方程为x?0.1cos(20?t??/4)m,求:(1)振幅、频率、角频率、;周期和初相;(2)t=2s时的位移、速度和加速度.

解:对照谐振动的标准方程x?Acos(?t??)可知: (1) A?0.1m,??20?,??(2)

?4,???1?10Hz,T??0.1s 2??xt?2?0.1cos(20??2?)?7.07?10?2m,

4?vt?2??0.1?20?sin(20???4)??4.44m?s?1,

at?2??0.1?(20?)2cos(20???4)??280m?a?2。

6-3设四个人的质量共为250kg,进入汽车后把汽车的弹簧压下5.0?10?2m.若该汽车弹簧共负担1000kg的质量,求该汽车的固有频率.

1F250?9.84?1??解:由k?,??4.9?10N?m2??x5?10?2k1?m2?4.4?104?1.11Hz。

10006-4一立方体木块浮于静水中,其浸入部分的高度为a. 今用手指沿竖直方向将其慢慢压下,使其浸入部分的高度为b,然后放手任其运动. 若不计水对木块的粘滞阻力,试证明木块的运动是谐振动,并求出振动的周期和振幅. 解:已知木块作简谐振动,其回复力必取:

f??kx的形式,回复力是重力和浮力的合力。

a b o x 木块的平衡条件为;m木g?Sa?水g?m木?Sa?水

以静浮时下底面所在位置为坐标原点,x轴向下为正, 当下底面有位侈x时木块所受回复力为:

41

f??S?水(x?a)g?m木g??S?水gx??kx

所以k?S?水g,?T?2?Sa?水m?2??2?kS?水ga g取刚放手时为初始时刻,则;x0?b?a,v0?0,A?x0?6-5在U形管中注入水银,其密度为?,高度为l,

v0?2?b?a。

管的截面积为S. 今使水银上下振动,不计水银与管壁

的摩擦, 求振动的周期. l 解:当水银面处于任意位置y时,整个水银受回复力

f??2y?sg??ky,?k?2?sg题6-5图

T?2?m2?lsl?2??2? k2?sgg6-6一质量为1.0kg的物体放置在平板上,平板下面连着一个弹簧. 现使平板

上下做谐振动,周期为0.5s,振幅为2cm. 求:(1)当平板到最低点时物体对平板的压力;(2) 若频率不变,振幅多大时可使物体恰好离开平板? (3) 若振幅不变,频率多大时可使物体恰好离开平板?

???解:由于物体作简谐振动,在任意位置时所受作用力为(mg?N)。N为平板对物体支撑力随位置不同而变。

(1)取坐标x轴向下为正,则mg?N?ma,在最低点时x?A

?a???2A??(2?2)?2.0?10?2?0.32?2(m?s?2) 0.5?N?mg?ma?12.9N,则物体对平板的作用力N'?12.96N

(2)由于物体跳离平板是N=0 ,所以mg?ma?m?2A,?A?(3)由mg?m?2A可得:??g?2 ?6.2?10?2m。

?1?2?2?g1?A2?9.8?3.52Hz

2.0?10?26-7一物体放置在平板上,此板沿水平方向作谐振动. 已知振动频率为2Hz,物体与板面的最大静摩擦系数为0.5. 问:要使物体在板上不发生滑动,最大振幅是多少?

解:?mamax??mg,?m?2A??mg;则:A??g?g?2??3.1?10m。 22?(2??) 42

6-8一水平放置的弹簧受到1N的力作用时伸长5.0?10?2m. 现在此弹簧的末端系一质量为0.064kg的物体,并拉长0.10m后放手任其振动, 试求此弹簧振子作谐振动时的周期、最大速度和最大加速度.

解:k?m0.064F1?1T?2??2??0.36s。 ;??20Nm?2k20?x5.0?10kk?1.77m?s?1amax?A?2?A?31.3m?s?2。 mmvmax?A??A6-9一质量为1.0?10?2kg的物体作谐振动,其振幅为2.4?10?2m, 周期为4.0s,当t=0时位移为2.4?10?2m. 求: (1) 在t=0.50s时物体所在的位置和物体所受的力;(2) 由起始位置运动到x??1.2?10?2m处所需的最短时间.

解:(1)?x0?2.4?10?2m,????0,又??2?2???? T42?x?2.4?10?2cos(t),当?t?0.5s时,x?1.7?10?2m,

2?f??kx???2mx??4.2?10?4N

?x?24(2)?x?1.2?10?2m时,cost??0.5;?tmun??,tmin???1.33s。

2A2336-10作谐振动的物体,由平衡位置向x轴正方向运动,试问经过下列路程所需的

时间各为周期的几分之几?(1) 由平衡位置到最大位移处;(2) 这段距离的前半段:(3) 这段距离的后半段.

解:(1)????(2)????(3)?????2,??????2???T;??t?? T?4T 12T 6?6,??t?,??t?????3?6-11两质点沿同一直线作同振幅、同频率的谐振动. 在振动过程中,每当它们经过振幅一半的地方时相遇,而运动方向相反. 求它们的相位差,并作旋转矢量表示之.

解:在一次完全完全振动中,对应于一个位置可有二个等值反向的速度。

?x?Acos(?t??),v?A?sin(?t??)

43

(a)设甲乙甲乙两质点在正方向相遇

对甲:cos(?t??1)?0.5,sin(?t??1)<0 得:?t??1?5?。 31? 3?t??2)?0.5,sin(?t??2)>0 得:(?t??2)?对乙:cos(?(?y??1)?(?t??2)?4?。 3(b)设甲乙两质点在负方向相遇。

对甲:cos(?t??1)??0.5,sin(?t??1)<0 ;得:?t??1?4?。 32?。 3?t??2)??0.5,sin(?t??2)>0 ;得:?t??2)?对乙:cos(?(?t??1)?(?t??2)?2? 36-12两个质点作同频率、同振幅的谐振动, 第一个质点的振动方程为

x1?Acos(?t??). 当第一个质点自振动正方向回到平衡位置时,第二个质点恰在振动正方向的端点,求第二个质点的振动方程和两振动质点的相位差.

解:由旋转矢量图得:

????2??1???2,??2??1??2;第二个振动方程为:x2?Acos(?t??1??2)

6-13原长为0.50m的弹簧上端固定,下端挂一质量为0.10kg的砝码. 当砝码静止时,弹簧的长度为0.60m. 若将砝码向上推,使弹簧缩回到原长,然后放手,砝码上下运动. (1) 证明砝码的上下运动为谐振动;(2) 求此谐振动的振幅、角频率和频率;(3) 若从放手时开始计算时间,求此谐振动的运动方程(正向向下).

解:本题证明在任意时刻作用在物体上的合力为:f??kx的形式即可。 (1)以物体平衡位置为原点,x轴竖直向下为正方向。设物体平衡时使弹簧伸

'长了x0。有:kx0?mg,

'')?mg??kx; 当物体处于任意位置时有:f??k(x?x0所以物体作简谐运动。

'(2)取放手时刻为初始时刻,则初位移为?x0,初速度为0。

'?A?x0?v0?2'?x0?0.1m;??k?m44

g??1?9.9s;???1.58Hz。 'x02?

(3)由初始条件可定出;???,所以方程为:x?0.1cos(9.9t??)

6-14如图所示,质量为10g的子弹以1000?m?s-1的速度射入木块并嵌在木块中,使弹簧压缩从而作谐振动. 若木块质量为4.99kg,弹簧弹性系数为

8?103N?m-1,求振幅.

解:由动量守恒定律:mv?(m?M)V得碰后速度

?V?11由机械能守恒:(m?M)V2?kA

20mv m?V得A?m?M2mvV??0.05m kk(m?M)6-15质量为0.10kg的物体作振幅为1.0?10?2m的谐振动,其最大加速度为(2)通过平衡位置时的动能;(3)物体在何处4.0m?s?2. 求(1)振动的周期;其动能与势能相等?

解:(1)因为:amax??2A;所以:??(2)Ek?(4)Ep?amax2??20s?1;T??0.314s A?1112(3)E?mVmmV2max?mA2?2?0.002J;ax?0.002J;

222121kx、Ep?Ek?kA2;当Ep?Ek时有: 22A2x????0.707?10?2m。

26-16当谐振子的位移为其振幅的一半时,其动能和势能各占总能量的多少?物体在什么位置其动能和势能各占总能量的一半?

解:(1)Ep?(2)Ep?Ek?121A211213kx?k()??kA?E;所以:Ek?E?Ep?E 2224244111E,所以:kx2?kA2;x??0.07A。 2246-17一个质点同时参加两同方向、同频率的谐振动,振动方程分别为

45

x1?6cos(2t??/6)cm,x2?8cos(2t??/3)cm,

试用旋转矢量法求合振动方程.

解:由旋转矢量图可得:

A?2A12?A2?62?82?10cm

???3?tg?16?0.403rad 8所以合振动方程为;x?10cos(2t?0.403)cm

6-18已知两个同方向、同频率的谐振动的振动方程分别为

x1?5cos(10t?0.75?)cm, x2?6cos(10t?0.25?)cm.

求:(1)合振动的振幅及初相; (2)若另有一同方向、同频率的谐振动方程为

x3?7cos(10t??3)cm, 则?3为多少时,x1?x3的振幅最大?又?3为多少时,x2?x3的振幅最小?

解:(1) A?2A1?A2?2A1A2cos(?1??2)?7.81cm

2??tg?1A1sin?1?A2sin?2?tg?111?84o48'

A1cos?1?A2cos?2(2)要使x1?x3的振幅最大,必须使两振动同相位: ?3??1?075? 合成振幅为: A?A1?A2?12cm

(3)要使x2?x3的振幅最小,必须两振动反相:?3??2??,?3?1.25? 合振幅为:A?A3?A2?1cm

6-19有两个同方向、同频率的谐振动,其合振动的振幅为20cm,合振动的相位与第一个振动的相位之差为30?. 若第一个振动的振幅为17.3cm,求第二个振动的振幅, 第一、第二两振动的相位差.

解:由旋转矢量图可知:A2?A2?A12?2AA1cos30o?10cm

46

???tg?1Asin30o?? oAcos30?A126-20示波管的电子束受到两个互相垂直的电场的作用,若电子在两个方向上的位移分别为x?Acos?t和xy?Acos(求在??0、??30?、??90?各?t??),情况下,电子在荧光屏上的轨迹方程.

解:两同频率、互相垂直的谐振动合成,其轨道方程为:

x2y22xy??cos(?2??1)?sin(?2??1) 2A1A2A12A2(1)?A1?A2?A,?1?0,?2?0代入上式得轨道方程为:

x2?y2?2xy?0;?x?y

(2)?A1?A2?A,?1?0,?2?30o轨道方程为:

x2?y2?3xy?12A 4(3)?A1?A2?A,?1?0,?2??2轨道方程为:x2?y2?A2

6-21一个弹簧振子的质量m?1.0kg,弹性系数k?900N?m?1,阻尼系数?=

10.0s?1. 当振子在周期性强迫力F?100cos30tN 的作用下做稳定受迫振动时,

其角频率和振幅极大值分别是多少?

解:(1)受迫振动达到稳定时有:?1??p?30s?1

(2)受迫振动达到稳定时其振幅为: A?当:dAF02?02??p?4??0, md?pF022?1?0时振幅取极大值可求得?p??0?2??26.5s,

?Ar?2m????202?0.177m

6-22如图所示,一个由两个密度均匀的金属米尺构成的T形尺可绕过上端点的水平轴左右自由摆动. 若摆角足够小,求摆动的周期.

47

解;T形尺的微小振动是复摆振动,T形尺绕轴O的转动惯量J由两部分组成:

12117ml222. Jo?J1?J2?ml?(ml?ml)?31212T形尺的质心C至点O的距离为lC,由质心定义可得lC?0.75l,则振动周期为:

T?2?Jo/2mglC?2?17l/18g?1.95s.

习题 7

7-1一声波在空气中的波长是0.25m,速度是340m?s-1. 当它进入另一介质时,波长变成了0.79m,求它在这种介质中的传播速度.

解:波在不同介质中传播时频率不变

?v1??1?,v2??2?。?v2??2?1074.4m?s?1 ?17-2已知波源角频率??12.56?102s,波速??380m?s-1,振幅A?1.0?10?2m,初相??0,试写出波动方程.

x???x?解:由波方程的标准形式:y?Acos??t???2???Acos??t??可得:

??????y?1.0?10?2cos12.56?102(t?x/380) m

7-3波源作谐振动,其振动方程为y?4?10?2cos240?tm,它所形成的波以x30m?s-1的速度沿一直线传播. (1) 求波的周期及波长;(2) 写出波动方程. 解:对照振动方程y?Acos?t可知:??240?s?1,A?4?10?3m (1) T?2???8.3?10?3s ,??vT?0.25 m

xx (2) 波动方程:y?Acos?(t?)?4?10?3cos240?(t?)m

v307-4一维谐波波源的频率为250 Hz,波长为0.1m,振幅为0.02 m,求:(1) 距波源1.0 m处一点的振动方程及振动速度;(2) 波的传播速度; (3) t=0.1s时的波方程,并作图.

xx解:(1)波动方程为:y?Acos2?(?t?)?0.02cos(250t?)m;

?0.1 48

dyx??10sin2?(250t?), dt0.1以x=1.0 m代入上式得: v振?y?0.02cos(500?t?20?);v振??10?sin(500?t?20?)

(2) c?????0.1?250?25m?s?1

(3) 以t=0.1 s代入波动方程有;y?0.02cos(50??20?x)

?试求t=0.1 s时,x=2 m处一点的位移、速度和加速度.

2?解:y?0.01cos(25t?x)?0.01cos(250?t?10?x)m

0.2v振?7-5一横波的方程为y?Acos2π??0.2m、v?25s?1,(ut?x).若A?0.01m、

dy??2.5?sin(250?t?10?x)m?s?1;a振?y''??625?2cos(250?t?10?x) dt将 t=0.1 s, x=2 m代入以上各式得:

y??0.01m;v振?0;a振?625?2m?s?2

π7-6波源的振动方程为y?6?10?2cost(m), 它所形成的波以2.0m?s-1的速

5度在一直线上传播. 求: (1) 距波源6.0 m处一点的振动方程;(2) 该点与波源的相位差;(3) 此波的波长.

?解:(1) ?y振?6?10?2cost m ,v?2.0m?s?1

5?x?y?6?10?2cos(t?),以x=6.0 m代入得:

52?3yx?6.0?6?10?2cos(t??)m

553(2) 该点的振动相位比波源落后?;

52?20 m .

?0.17-7 波源作谐振动,周期为1/100 s,并以它经平衡位置向正方向运动时为时

(3) ??c?间起点. 若此振动以u?400m?s-1的速度沿直线传播, 求距波源为800cm处的振动方程和初相. 又距波源为990 cm和1000 cm处两点之间相位差为多少?

1解:由题意知波源的振动方程为:y?Acos(200?t??)

2(1)距波源为800cm的振动方程为:

1xy?Acos(?t???2?)?Acos(200?t?4.5?),

2? 49

初相?0??4.5?. (2) ???2?x2?x1??0.05?.

7-8一平面波在介质中以速度u?20m?s-1沿x轴的负方向传播. 已知在传播路径上某点A的振动方程为y?3cos4πt(cm). (1) 以A点为坐标原点,写出波动方程;(2) 以距A点5m处的B点为坐标原点,写出波动方程;(3)写出传播方向上B点、C点、D点的振动方程(各点间的距离参看图示). u 5m 9m 8m CBA D x 题7-8图

?解:(1) y?3cos(4?t?x);

5?(2) y?3cos(4?t???x);

513(3) B点振动方程为:y?3cos(4?t??);C点:y?3cos(4?t??);

55 D点:y?3cos(4?t??)

97-9 两平面波源A、B振动方向相同,相位相同, 相距0.07 m,它们所发出的波的频率??30Hs,波速 P 为u?0.5m?s. 求在与AB连线成30夹角的直线上 -1? 3 m 距A为3 m的P点处两列波的相位差. P点初相

30o 是多少?设两波的振幅相同,初相均为零. A 0.07m B 题7-9图

解:由图知:AP、BP》AB;

?AP?BP?ABcos30o?0.07AP?BP3?7.27?. m ;???2?2??47-10有一波在介质中传播,其波速u?103m?s-1振幅A?1.0?10, 频率m??103Hz. 若介质的密度为800kg?m-3,求:(1) 该波的能流密度;(2) 1分钟内垂直通过一面积S?4?10?4m2的总能量. 解:(1) I?11?v?2A2??800?103?(2??103)2?(1.0?104)2?1.58?105w?m?2 22 (2) W?ISt?1.58?105?4?104?60?3.77?103J

50

热量传递给系统,同时系统对外作功126J. 如果系统从状态C沿另一曲线CA回到状态A,外界对系统作功52 J,则此过程中系统是吸热还是放热?传递多少热量?

解:系统经ABC过程所吸收的热量及对外所作的功分别为:

QABC?326J,WABC?126J

由热力学第一定律可得由A到C过程中系统内能的增量

?EAC?QCA?QABC?WABC?200J

从C到A,系统吸收的热量为;QCA??ECA?WCA??252J

上式中负号表示系统向外界放热252J,这里要说明的是由于CA是一末知过程,上述求出的放热是过程的总效果,而支其中每一个微小过程来讲并不一定都是放热。 pp (atm) 4 B CA D 3 C

2

A E 1 B OV(l)OV(l)

题8-9图题8-10图

8-10 如图所示,一定量的理想气体经历ACB过程吸热200 J,则经历ACBDA过程时吸热又为多少?

解:由图中数据有pAVA?pBVB,则A、B两状态温度相同,故ABC过程内能变化?EABC?0,由热力学第一定律得系统对外界作功

WABC?QABC??EABC?QABC?200J

在等体过程BD及等压过程DA中气体作功分别为

WBD??pdV?0;WDA??pdV?pA(VA?VD)??1200J

则在循环过程ACBDA中系统所作的总功为

W?WACD?WBD?WDA??1000J,负号表示在此过程中,热量传递的总效果为放

热。

8-11 2mol理想气体的体积在300K的温度下从4?10?3m3等温压缩到

1?10?3m3,求在此过程中气体作的功和吸收的热量.

解:等温过程:?E?0,

由热力学第一定律知;QT?WT。其所作的功和吸收的热量为:

56

WT?QT?nRTlnV2??6.91?103J V1 p(atm)

8-12 如图所示,使1摩尔的氧气(1) 由a等温地变到b;(2)由a等容地变到c,2 a 变b,再由c等压地变到b。试分别计算所 作的功和吸收的热量。1 cb

O 22.4 44.8 V(l)题 8-12图

解:(1)等温过程:?E?0,Wab?Qab?pbValnVb?31.5?102J VaiiWac?0,(2)等容过程:Qac?nR(Tc?Ta)?(nRTc?nRTa)??56.4?102J

22(3)等压过程:Wcb?pc(Vb?Vc)?22.7?102J

Qcb?ni?2i?2(Tb?Tc)?(pbVb?pcVc)?79.4?102J 22?Qacb?Qac?Qcb?22.7?102J,Wabc?Wcb?22.7?102J

8-13一定量的的氮气,温度为27?C,压力为1 atm,今将其绝热压缩,使体积变为原来的1/5求压缩后的压力和温度。

解:由绝热方程p1V1?p2V2?,得氮气绝热压缩的压强为:

p2?(V1?)p1?51.4?9.51atm. V2??????1??V1??1??1又由:V1T1?V2,得:T2???V?2T1?571k

8-14 试证明1 mol理想气体在绝热过程中所作的功为:

A?R(T1?T2).

??1V2证:

W??pdV??V1VV2V1p1V1??1?V2????1?dV?pVVdV?pVV11?11??V1V????1?V1

?p1V1?pV2R(T2?T1)???1??1

57

8-15 0.32 kg的氧气作如图所示的循环,设V2?2V1,求循环效率 p p

T1?300Ka等温

等ba 容绝 d热 c T2?200K c b OV1V2VO V1V2V 题8-15图题8-16图 解:

W?WAB?WCD?nRT1ln(V2/V1)?nRT2ln(V1/V2)?nR(T1?T2)ln(V2/V1)?5.76?103JQ?QAB?QDA?WAB??EDA?nRT1ln(V2/V1)?nCV(T1?T2)?3.81?10J4

由此得到该循环的效率为:??W/Q?15%.

8-17一卡诺热机的低温热源温度为7?C,效率为40%,若要将其效率提高到50%,求高温热源温度提高多少度? pp

b c pd d d pc c a paa OV2V3V1V OVcVbV

题8-18图题8-19图

8-18 如图所示为理想的狄赛尔(Diesel)内燃机循环过程,它由两绝热线(ab,cd),一等压线(bc)及一等容线(da)组成,试证明此热机的效率为

??1??V3/V2???V1/V2???1??1?V3??1???V2?

证:因为:Q吸?Qbc,Q放?Qda

58

由??1?Q放Q吸?1?QdaQbc?1?nCV(Td?Ta)T?Ta?1?d

nCp(Tc?Tb)?(Tc?Tb)??1?V1?TTV?其中:c?1,b????TVTbV2a?2??TdV1??1?TcV3??1

解上各式,可证得:??1??V3/V2???V1/V2???1??1?V3??1??V?2?

8-19 汽油机可近似地看成如图所示的理想循环. 这个循环也叫做奥托(Otto)循环,其中de和cb是绝热过程,(1)证明此热机的效率为:??1?Te?Tb, 式

Td?Tc中Tb、Tc、Td和Te分别为状态b、c、d和e的温度. (2) 利用TV??1?C,上述效率公式可写成??1??Vc/Vb???1.

解:求证方法与上题相似

(1) 该循环仅在CD过程中吸热,EB过程中放热,则热机效率为:

??1?QEBQCD?1?nCV(TE?TB)T?TB?1?E

nCV(TD?TC)TD?TC(2) 在过程BC和DE中,分别分别应用绝热方程:TV??1?C有

??1??1??1??1;TEVB由此两式可得: TBVB?TCVC?TDVCTE?TB?VC?????TD?TC?VB????1将此结果代入(1)中,即可得:

??1?(VC/VB)??1,得证。

习题 9

9-1 已知分布概率

?(x,y)dxdy?xydxdy,

其中0?x?a,0?y?b. (1) 试将概率密度函数??x,y?归一化. (2) 求区域

x?x?dx内的概率.

abab解:(1)令?(x,y)?Axy, 由???(x,y)dxdy?A??xydxdy?1,得归一化常数

0000 59

A?1ab???xydxdy004, 22ab4xy. 22ab (2) 区域x?x?dx内的概率为

所以?(x,y)??b?2dW????(x,y)dy?dx?2xdx.

a?0?9-2已知概率密度为?(x)?ae?ax, 其中常数a?0, 0?x??. 求x,

(x?x)2.

??x2和

解:x??x?(x)dx?a?e?axxdx?00?221, a?x??x?(x)dx?a?e?axx2dx?002, a2122所以x?,而后(x?x)2???x?x??(x)dx?x2?x2?2.

aa02?9-3 在容积为20l的容器中装有质量为2g的氢气. 若氢气的压强为300 mmHg,

氢气分子的平均平动能是多少?

解:氢气分子的平均平动动能:?k?3kT/2?3pVM/(2mN0)?1.99?10?21J 9-4 温度为0?C和100?C时,空气分子的平均平动能是多少? 解:分子在0oC和100oC时平均平动动能分别为:

?1?3kT1/2?5.56?10?21J;?2?3kT2/2?7.72?10?21J

9-5 目前,在实验室中已经获得了压强为10?10mmHg的所谓“真空”. 试问:在

27?C的温度下,这样的“真空”中每立方厘米内有多少个分子?

解:由气体压强公式:

p?nkT可得分子数密度

p10?10?1.33?10212?36?3n???3.22?10m?3.22?10cm ?23kT1.38?10?3009-6 已知一定质量的空气在27?C的温度下的体积为10l. 若压强不变,当温度

60

?2?546.1nm的汞灯绿光的1/4波片的最小厚度为多少?

解;1/4波片的最小厚度为d应满足no?ned??4而方解石晶体中的o光和

e光的折射线分别为no?1.658和ne?1.486。故对?1和?2的1/4波片的最小厚度分别为:d1??1?857nm;d2??2?794nm。

4no?ne4no?ne71

为127?C时,气体体积为多少?

解:视空气为理想气体,由于体积不变,则由

T2V1?13.3l T1V1V2?可得: T1T2V2?9-7 标准状况下二氧化碳气的分子数密度是多少? 解:视CO2为理想气体,由pV?MRT可得: Mmol??MpMmol??1.96kg?m?3 VRT9-8 温度为0?C和100?C时,理想气体分子的平均平动动能各为多少?欲使

分子的平均平动动能等于1eV,气体的温度需多高? 解:分子在0oC和100oC时平均平动动能分别为:

?1?3kT1/2?5.56?10?21J;?2?3kT2/2?7.72?10?21J

由于1eV?1.6?10?19J,因此,分子具有1eV平均动能时,气体温度为:

T?2?k/3k?7.73?10K,这个温度约为7.5?10C

33o9-9有N个质量均为m的同种气体分子,Nf(?) 它们的速率分布如图所示.(1) 说明曲线与a 横坐标所包围面积的含义;(2) 由N和?0

求a值;(3) 求在速率?0/2到3?0/2间隔 0 ?02?0? 内的分子数;(4)求分子的平均平动动能。题9-9图

解:(1)由于分子所允许的速率在0到2v0的范围内,由归一化条件中曲线下的面积

2v0S??0Nf(v)dv?N,即曲线下面的面积表示系统总分子数N。

(2)从图中可知,在0到v0区间内,Nf(v)?av/v0;而在v0到2v0区间关

Nf(v)?a,则利用归一化条件有N??v02v0avdv??adv

v0v00 61

得:a?2N/3v0.

(3)速率在v0/2到v0/3间隔内的分子数为;

?N??v02v00av2dv??adv?7N/12

v0v03v(4)分子速率平方的平均值按定义这:

v??vdN/N??v2f(v)dv,故分子的平均平动动能为:

002v0a?312121?v0a22??k?mv?m?vdv?vdv????36mv0 0Nvv0N22?0??2?2?9-10求温度为127?C时的氢气分子和氧气分子的平均速率、方均根速率及最概然速率.

解:氢气分子的平均速率、方均根速率、及最概然速率分别为:

v?1.6RT8.13?400?1.6?2.06?103m?s?1 ?3Mmol2?10RT8.31?4003?1?1.73?2.23?10m?s ?3Mmol2?10RT8.31?400?1.41?1.82?103m?s?1 ?3Mmol2?10v2?1.73vp?1.41氧气分子的平均速率、方均根速率、及最概然速率分别为:

v?1.6RT8.31?400?1?1.6?516m?s ?3Mmol32?10RT8.31?400?1.73?558m?s?1 ?3Mmol32?10RT8.31?400?1?1.41?454m?s Mmol32?10?3v2?1.73vp?1.419-11在1 atm下,氮气分子的平均自由程为6?10?6cm. 当温度不变时,在多大压力下,其平均自由程为1 mm? 解:由平均平均自由程计算公式:??kT2?dp212?d2n,式中n?p代入得: kT??

,由题意温度不变,即T1?T2?T

62

故由上式得:p1?1?p2?2,则平均自由程为1mm时的压力为:

p2??1p1?6.06Pa ?29-12收音机所用电子管的真空度约为1.0?10?5mmHg,试求在27?C时单位体积中的分子数及分子的平均自由程(设分子的有效直径d?3.0?10?8cm).

解:由p?nkT得单位体积中的分子数为:n?分子的平均自由程:??12?dn2p?3.22?1017m?3 kT?kT2?dp27.77m

9-13若氖气分子的有效直径为2.04?10?8cm,问在温度为600 K,压力为1 mmHg时氖气分子一秒钟内的平均碰撞次数为多少? 解:一秒内的平均碰撞次数为:

p??1.60RTZ?2?dnv?2?dkT?Mmol?22???2.38?106s?1 ??1 9-14已知分子的平均平动动能??m?2. 试将麦克思韦速率分布定律写成下

2面的能量分布定律dN?N2(KT)?3/2?1/2e??/KTd?. π32?mv22kT?m?解:麦克思韦分子速率分布定律为:dN?N4???e?2?kT?1由于??mv2,则d??mvdv得:

2dN?N2

2?2?3232?kT?12?3?2??13??2?m??kT2d???1kT2??kT?2??Nmved? ??ev??2?mv????2?32?N??kT??kT??ed?

19-15利用上面的能量分布定律,证明分子的最可几平动动能为KT.

2证:由上题的能量分布定律得能量分布函数

???dN222f(?)??(kT)?ekT

Nd??31? 63

??311????1??d?f????21?(kT)2??2ekT??2ekT??0 则:

d?kT??2?121??11/22?1???2???0,?1???0 ?2kT?2?1/2kT????1?21?2e??kT??kTe??kT?e??kT得最可机平动动能为:?p?1kT 239-16利用9-14的能量分布定律,证明分子的平均平动动能为KT.

21提示:分子的平均平动动能为????dN.

N?证:???2N0?dNN??0?2N?1/23/2?(kT)Ne??kTd?1?3/2?(kT)2??0?3/2e??kTd?

?2??0(?kT)3/2d(?kT)kT?2?(kT)?x3/2e?xd?(令

0??kT?x)

??3?3(kT)????kT得证。 ??4?2 习题10

10-1如图10-1所示,S1和S2是两个相距4.0 m的 同相点光源, 以同样的功率发射波长为1.0 m的电磁波.

y/m S2 如果一个探测器从S1处沿着x轴向右运动,它可以发现x/m S1多少个加强点?这些加强点距离S1多远?题10-1图

解:显而易见, X轴上一点到S2的距离为r?16?x2,而光程差为

??r?x?k?,

16?k2?0. 所以x?2k当k?1,2,3,4时,k?7.5,3,1.17,0.

10-2在洛埃镜实验中,将屏P紧靠平面镜M的右边缘L点放置,如图所示.已知单色光源S的波长??720nm,求平面镜右边缘L到屏上第一条明纹的距离.

64

2mm d

L ML

20cm 30cm P

题10-2图题10-3图 解:设?x为双缝干涉中相邻明纹(或暗纹)之间的间距,L到屏上第一条

?x1d'明纹距离为:x????4.5?10?5m

22d10-3如图所示,利用空气劈尖测细丝直径. 已知入射光波长??589.3nm,玻璃片长度L?2.888?10?2m,测得30条条纹的总宽度为4.295?10?3m,求细丝的直径d.

解:应用劈尖干涉公式d?细丝直径为d??2ndL,由分析知,相邻条纹间距b?L?5.57?10?5m

?x,则N?1?2n2bL???N?1?2n2?x10-4有一玻璃劈尖, 玻璃的折射率为1.5,劈尖角??5?10?5rad. 用单色光垂直照射,测得干涉条纹宽度l?3.64?10?3m,求此单色光的波长.

解:????2vl10-5 在牛顿环实验中,当用波长为589.3 nm的钠黄光垂直照射时,测得第

,????2nl?5460Ao

一和第四暗环的距离为?r?4.0?10?3m;当用波长未知的单色光垂直照射时,测得第一和第四暗环的距离为?r'?3.85?10?3m,求该单色光的波长. 解;暗环的半径?r?kR???r?(4?1)R?;?r'?(4?1)R?'

?r'?故:?r?'?,所以:?'?5460A0

10-6 用波长为589.3 nm的钠黄光观察牛顿环,测得某一明环半径为1.0 mm,而其外第四明环半径为3.0 mm,求平凸透镜凸面的曲率半径. 解:设题中所述两个明环分别对应k级和(k+1)级明纹,则有:

1?1???rk??k??R?;rk?1???k?4???R?

2?2???rk2?4?rk2?3.39m 解上述两式可得:R?4? 65

10-7当牛顿环装置中的透镜与玻璃之间的空间充以某种液体时,第十明环的直径由14 mm变为12.7 mm, 试求这种液体的折射率.

解:当透镜与玻璃之间为空气时,k级明纹的直径为:

dk?2rk?21???k??R?

2??当透镜与玻璃之间为液体时,k级明纹的直径为:

1?R??dk'?2rk'?2?k??

2?n2??dk解上述两式得:n2???d'?k A ?1 B ???1.22 ??2 S ? 空气n1 n2 d 空气n3

n d

C 题10-8图题10-9图

10-8如图所示,由S点发出的??600nm的单色光,自空气射入折射率为n=1.23的透明物质,再向射入厚度d=1.0 cm,入射角??300,且SA=BC=5 cm,求:(1) 折射角?1为多少?(2) 此单色光在这层透明物质里的频率、速度和波长各为多少?(3) S到C的几何路程为多少?光程又为多少?

解:(1) 由析射定律

sin??n可得: sin?1?sin30o?sin???1?arcsin???arcsin??1.23n????o??24 ??(2) 单色光在透明介质中的速度vn,波长?n和频率?分别为;

v?c?2.44?108m?s?1;?n??488mm;???5.0?1014Hz nn?(3) S到C的几何路程为: vn?SC?SA?AB?BC?SA?d?BC?0.111m cos?1S到C的光程为:???niDi?SA?1?AB?n?BC?1?0.114m

10-9如图所示,用白光垂直照射厚度为d=400 nm的薄膜,若薄膜的折射率

66

为n2?1.40且n1?n2?n3,问反射光中哪种波长的可见光得到加强?

解:因n1?n2?n3故薄膜上下面面反射光都不发生半波损失。又i?0 所以反射光程差:??2n2d?k?为加强 即:??2n2d,当k=1、2、3时,?1?11.2?103Ao(属于红外区) k;?3?3.7?103Ao(属于紫外区) ?2?5.6?103Ao(属于黄光)

10-10折射率n3?1.52的照相机镜头表面涂有一层折射率n2?1.38的MgF2增透膜,若此膜仅适用于波长??550nm的光,则此膜的最小厚度为多少?

解:因干涉的互补性,波长为550nm的光在透射中得到加强,则反射中一

?定减弱,两反射光的光程差?2?2n2d,因干涉相消条件?2?(2k?1)得:

2d?(2k?1)?4n2,取k=0,则dmin?99.3nm

10-11一单缝宽度a?1.0?10?4m,透镜的焦距为f?0.5m,若分别用

?1?400nm和?2?760nm的单色光垂直入射,问它们的中央明纹宽度各为多少?

解:对?1?400nm,l01?对?2?760nm,l02?2f?1?4?10?3m a2f?2?7.6?10?3m a10-12一单色光垂直照射于一单缝,若其第三级明纹位置正好和600 nm的单色光的第二级明纹位置一样,求前一种单色光的波长.

解;因为衍射角与波长成正比,当两波长的衍射角相等时,明纹位置一样,

5??'7?''即:asin?2?(2k?1)?;asin?3?(2k?1)?

2222?由:sin?2?sin?3可得:?'?5??428.6nm 710-13已知单缝宽度a?1.0?10?4m,透镜焦距f?0.5m,用?1?400nm和

?2?760nm的单平行色光分别垂直照射,求这两种光的第一级明纹离中心的距离

以及这两条明纹之间的距离. 若用每厘米刻有1000条刻线的光栅代替这个单缝,则这两种单色光的第一级明纹分别距屏中心多远?这两条明纹之间的距离又是多少?

67

解:(1) 单缝:明纹位置:x?ftg??fsin??(2k?1)对?1?400nm,x1??2af

3?1f?3mm 2a3?2f?5.7mm 2a对?2?760nm,x2?所以:?x?x2?x1?2.7mm

(2) 光棚:明纹位置:x?ftg??fsin??对?1?400nm,x1?k?f a?b?1a?bf?2cm

对?2?760nm,x2??2a?bf?3.8cm

所以:?x?x2?x1?1.8cm

10-14 在双缝干涉实验中,两缝的间距为0.3mm.当用平行单色光照射双缝时,正负第5级暗纹的间距为22.78mm. 已知屏幕到双缝的距离是1.20m,求入射光波长. 它是什么颜色的光?

D?解法一:在双缝干涉中,屏上暗纹位置xk?(2k?1), 因此,正负第5级暗

d2D?纹的之间的距离s?x5?x?5??11??9?????d2,

sd所以, ???569.5nm.

10D注意,s?2x5.

解法二:因双缝干涉的明(或暗)条纹是等距的,相邻暗纹的间距?x?注意到正负第5级暗纹之间所包含的相邻条纹间隔数为10, 故?x?D?. ds,所以 10?xsd??569.5nm, 黄光. D10D10-15一双缝装置的一个缝被折射率为1.40的薄玻璃片所遮盖,另一个缝被折射率为1.70的薄玻璃片所遮盖. 在玻璃片插入以后,屏上原来的中央极大所在点现在为第五级明纹. 假定??480nm,且两玻璃片厚度为d,求d.

??d解:插入介质前的光程差?1?r1?r2?k1?,

插入介质后的光程差?2???n1?1?d?r1????n2?1?d?r2??k2?,

68

光程差的变化为:?2??1??n2?n1?d??k2?k1???5? 所以:d?5?8.0?m n2?n110-16当瞳孔的线度为D=2 mm时,对于波长550 nm的绿光最小分辨率为多少?

解:由圆孔衍射得:?0?1.22?D?3.35?10?4rad

10-17某天文台反射式天文望远镜的通光孔径为2.5 m,试求能被它分辨的双星的最小夹角,已知波长为550 nm.

解:?0?1.22?D?2.684?10?7rad

10-18 一光栅在2.45 cm中有15000条窄缝,对某一波长的光,测得第一级衍射角为13?40?,求此光的波长.

解;由光栅公式:(a?b)sin??k?可得:

??a?bsin?k?400nm k10-19用每一毫米内有500条刻痕的平面透射光栅观察波长??589.0nm钠光,问光线垂直入射时,最多能看到几级光谱?

解:由光栅公式及???2时,k取得极大值有:

kmax??a?b?sin90o??3.4(k取3),即最多能看到第三级光谱。

10-20 已知波长为0.296 nm的X射线投射到一晶体上,所产生的第一级衍射线偏离原射线方向31.7?,求相应于此衍射线的原子平面之间的间距.

解:由布拉格公式:2dsin??k?可得

d?k??5.42A0 2sin?10-21如果迈克耳逊干涉仪中的M2反射镜移动距离0.233 mm,则数得的条

纹移动数为792,求所用光波的波长.(关于迈克耳逊干涉见15-1节)

解:由?d??k?2可得:??2?d?588.4nm ?k 69

10-22以波长为0.110 nm的X射线照射岩盐晶面,测得在X射线与晶面的夹角为13?30?时获得第一级极大的反射光,问:(1) 岩盐晶体中晶面的间距d多大?(2) 如以另一束待测的X射线照射岩盐晶面,测得X射线与晶面的夹角为

17?30?时获得第一级极大的反射光, 则待测的X射线的波长是多大?

解:由布拉格公式可得:

2dsin?'k?o?1066Ao ?2.76A;(2) ?? (1) d?k2sin?10-23 两偏振片的方向成30o夹角时,透射光强为I1,若入射光不变而使两偏振片方向夹角变为45?,则透射光强将如何变化?

解:由马吕斯定律有:I1?Io(cos30o)2;I2?Io(cos45o)2

cos45o22)?I1 所以:I2?I1(3cos30o10-24使自然光通过两个方向相交60?的偏振片,透射光强为I1. 今在这两个偏振片之间再插入另一偏振片,它的方向与前两个偏振片均成30o角,则透射光强为多少?

解;由马吕斯定律有:I1?I2(cos60o)2,I4?I3(cos300)2,I3?I2(cos30o)2 所以:I4?COS30???COS60?O4O2I1?2.25I1

10-25一束光是自然光和平面偏振光的混合,当它通过一偏振片时发现透射光的强度取决于偏振片的取向,其强度可以变化5倍,求入射光中两种光的强度各占总入射光强度的几分之几.

解:设入射混合光强为I,其中线偏振光强为xI,自然光强为(1-x)I.。按题意旋转偏振片,则有:

?1?最大透射光强:Imax???1?x??x?I

?2??1?最小透射光强:Imin???1?x??I

?2?按题意:

Imax112?5则有:(1?x)?x?5?(1?x),所以x? Imin22310-26试计算用方解石晶体制成的对波长分别为?1?589.3nm的钠黄光和

70

?2?546.1nm的汞灯绿光的1/4波片的最小厚度为多少?

解;1/4波片的最小厚度为d应满足no?ned??4而方解石晶体中的o光和

e光的折射线分别为no?1.658和ne?1.486。故对?1和?2的1/4波片的最小厚度分别为:d1??1?857nm;d2??2?794nm。

4no?ne4no?ne71

本文来源:https://www.bwwdw.com/article/kz3d.html

Top